PLEASE HELP ME WITH THIS LAST QUESTION PLEASE

Answers

Answer 1

Answer:

Is the answer -3x-y. If it is thank me plz.

Step-by-step explanation:


Related Questions

need answer asap Whole numbers: ___ + ___ + ___ = _____ cups of soup mix.

Answers

Answer:

1+2+3=6 lol jk i dont have the numbers that they gave you

Step-by-step explanation:

En la figura, la suma de las medidas p + q + r + s =/In the figure, the sum of the measures p + q + r + s =

And if someone know the explanation please write it​

Answers

Answer:

I think its 300

Step-by-step explanation

This is from my understanding, so I may be a bit wrong.

1. First observe the amount of triangles that form, you have two main triangles and one isosceles triangle that forms in the middle by how these are joined together.

2. Using this idea we will come up  with the equations of  each triangle that has been formed.

3. We can observe that almost all angles are named with a variable except the one in the middle, being an isosceles we will name the missing variable y.

4. Triangle 1(left side) or T1 equals 180 degrees but what angles form this triangle, these would be p+q+y = 180, T2(right side) = r+s+y = 180

5. After creating these two triangles we notice that we have not taken into account the triangle in the middle, we are given 1 angle of this isosceles. There is a mathematical property which I don't remember the name, but essentially the angle below is the same as the angle above, this being 120 degrees. This will result in T3 = 2y + 120 = 180

6. Solve for y, which gives us y=30, *simple triangle math you know*

7. Finally we fix our equations for p+q+r+s, essentially T1 and T2 will transform into p+q = 180-y and r+s = 180-y, we add them up forming p+q+r+s = 2(180-y)

8. Solve the equation p+q+r+s = 360 - 2y which when adding y becomes p+q+r+s = 360-60 = 300

Notes: when doing these exercises it is important to take into account every triangle, the isosceles triangle is very vital for solving it, as it gives us a way for making an equation that is single handedly dependent on y a constant we can find and use. A good approach for these issues would be writing every piece of information as separate sets and then trying to join them using algebra.

Two equations are shown: I will give 40 pts if someone answers


Equation 1: (x – 12) = 12

Solve each equation.

Answers

Answer:

x and y both equal 24

If the pth term of an A.P. is q and the qth term is p. Prove that its nth term is (p+q-n). ​

Answers

Given:

pth term of an AP = q

qth term = p

Prove:

nth term of A.P. is (p+q-n). ​

Proof:

We know that,

nth term of an AP (an) = a + (n - 1)d

Hence,

⟹ a + (p - 1)d = q

⟹ a + pd - d = q

⟹ a = q - pd + d -- equation (1)

Similarly,

⟹ a + (q - 1)d = p

Substitute the value of a from equation (1).

⟹ q - pd + d + qd - d = p

⟹ qd - pd = p - q

⟹ - d(p - q) = p - q

⟹ - d = 1

⟹ d = - 1

Substitute the value of d in equation (1).

⟹ a = q - p( - 1) + ( - 1)

⟹ a = q + p - 1

Now,

an = q + p - 1 + (n - 1)( - 1)

⟹ an = q + p - 1 - n + 1

⟹ an = p + q - n

Hence, Proved.

I hope it will help you.

Regards.

Step-by-step explanation:

ANSWER

pth term = q

a+(p−1)d=q

qth term = p

a+(q−1)d=p

Solving these equations, we get,

d=−1

a=(p+q−1)

Thus,

nth term = a+(n−1)d=(p+q−1)+(n−1)×(−1)=(p+q−n)

Eric is trying to save $37 to buy a gift for his mother. Right now he has $5. He can save $10 a week from mowing lawns and $5 a week from his allowance. What is the least number of weeks that Eric will need to save to have enough money for the gift?

Answers

Answer:

3 weeks. 5+10=15

because in 2 weeks he will have 30 dollars plus the 5 dollars he already has is 35 dollars but, that is not enough he needs one more week.

Help me solve this problem please please

Answers

Answer: D. x > -1

Step-by-step explanation:

A dolphin swam7 1/2 miles in 3/8 hour. what is its average speed in miles per hour?

Answers

Answer:

20 mph

Step-by-step explanation:

To find miles per hour you divide the number of miles by the number of hours

 

(7 1/2)/(3/8) = (15/2)(8/3) = 20 mph

Please help me with this. This is my last one and when you are answering PLEASE SHOW STEPS FOR BRAINSLT. THIS IS DUE TOMORROW

Answers

X= -2

Explanation: the photos are in order :)

Answer:

x = -2

Step-by-step explanation:

3/5 +3/10x = 4/5 + 2/5x

Step 1. Change every fraction into the same denominator

6/10 + 3/10 x = 8/10 + 4/10x

Step 2. Subtract 3/10x

6/10 = 8/10 + 1/10x

Step 3. Subtract 8/10

-2/10 = 1/10x

x = -2

The graph shows the number of hours that Tammy spends typing for work, x, and the amount of pay that she earns, y.

A graph titled Tammy's Pay with number of hours on the x-axis and pay in dollars on the y-axis. A line goes through points (2, 18) and (8, 42).

What does the y-intercept represent?

Answers

The y-intercept represents "How much Tammy is getting paid for each hour she works".

Given that the graph shows the number of hours that Tammy spends typing for work, x, and the amount of pay that she earns, y.

And, a graph titled Tammy's Pay with a number of hours on the x-axis and pay in dollars on the y-axis. A line goes through points (2, 18) and (8, 42).

How to find what the y-intercept represents?

If y, is the amount she gets paid then the line on the graph shows the ratio of how much she gets paid for each hour of work.

Learn more about y-intercept on: https://brainly.com/question/16009543

#SPJ2

Answer:

Answer is A

Step-by-step explanation:

here's the image for the question I have:

Answers

Answer:

c

Step-by-step explanation:

Answer:

C::10:15

Step-by-step explanation:

It is C because he had 15 hours of first aid and 10 hours of cpr and the question says CPR TO FIRST AID therefore the number of CPR training hours will come before the number of First Aid training hours. If it would have asked you to reduce the ratio it would have been 2:3 because you would have had to divide both sides by 5 resulting in the smallest possible ratio.

HOPE THIS HELPS YOU OUT!

(x2 – 2x – 2)(3x2 - 4x +4)

Answers

Answer:

10-4x

Step-by-step explanation:

The answer to the equation is 10-4x

Albert's mom has 2,523 reward points from shopping at an online store. She uses the points to buy gift cards for free cups of coffee. Each gift card costs 75 points. What is the greatest number of gift cards Albert's mom can buy? How many reward points will she have left? Use the drop-down menus to choose the numbers that make the statement true.

Answers

Answer:

Maximum number of gift cards she can purchase = 33

Reward points left with her = 48

Step-by-step explanation:

Let Albert's mom buys the number of gift cards = x

Points for each gift card = 75

Total points consumed to buy x gift cards = 75x

If total points with Albert's mom = 2523

So the remaining points with her = (2523 - 75x)

Inequality representing the remaining points will be,

(2523 - 75x) ≥ 0

75x ≤ 2523

x ≤ [tex]\frac{2523}{75}[/tex]

x ≤ 33.64

Maximum number of cards she can buy = 33

Therefore, reward points with her = 2523 - (75 × 33)

                                                         = 2523 - 2475

                                                         = 48

12⁴x 9³ x 4
6³ x 8² x 27​

Answers

Answer:

12⁴ × 9³ x 4 = 60,466,176

6³ × 8² × 27 = 373,248

Step-by-step explanation:

You could use the expanded form to calculate the answer

12⁴ = 20,736

9³ = 729

4 = 4

20,736 × 729 × 4 = 60,466,176

6³ x 8² x 27​

6³ = 216

8² = 64

27 = 27

216 × 64 × 27 = 373,248

1 x 1
2 x 2
4 x 4
16 x 16
200 x 19.56

Answers

Answer:

we should solve it or what should we do...

Which expressions are equivalent to 46 ?

Answers

Answer:

frus

ggyfhhfgggdg the nearest whole number is the nearest whole thing that is the nearest whole number is the nearest whole number is the nearest whole number is the nearest whole number is the nearest whole number is the nearest whole number is the nearest whole number is the nearest whole number is the nearest whole number is the nearest whole number is the nearest whole number is the nearest whole number is the nearest whole number is the nearest whole number is the nearest whole number of the nearest and the product of the nearest and we are all in this together to to 21211111111wwethkuff

Plz help quick >_< What is the best estimate for 81.9 x 4.6?
O A) 100
OB) 360
OC) 400
OD 8,000

Answers

Answer:

B.

Step-by-step explanation:

When you multiply the two decimals you get 376.74 which is closer to 360 than 400

Answer:
it's b
Step-by-step explanation:
I took the test

Is the following figure a parallelogram? Why?
P
O No, opposite angles are congruent.
O Yes, opposite sides are congruent.
O No, opposite sides are congruent.
O Yes, opposite angles are congruent.

Answers

Yes because opposite sides are congruent

I need help if you can.

Answers

Answer:

I cant see the equation that well, but i got 5/16

Step-by-step explanation:

Hope this helps!

Answer:

I got C) 1 and 1/4

Step-by-step explanation:

-0.5 + 0.75 = 0.25

1/5 = 0.20

0.25 divided by 0.20 would be over 1 because a fraction divided by another fraction brings the number while a whole number divided by a fraction would bring the number down.

0.25 divided by 0.20 = 1.25

Find the value of the expression below.
15 + 5 × 3
-------------- (fraction)
3² + 6

5
4
2

Answers

Answer:

2

Step-by-step explanation:

15 + 5 × 3

________

  3² + 6

For this question, we need to use PEMDAS to solve the question correctly. PEMDAS stands for "parentheses, exponents, multiplication, division, addition, subtraction". In this question, for the numerator, we see 5 x 3 so that's multiplication. So we multiply those together to get 15. In the denominator, there's an exponent which is 3². We have to find the square of 3 so that we can simplify the denominator as well. That simplifies to 9:

(15+15)/(9+6)

Next, we just have to add the 15+15 in the numerator together and 9+6 in the denominator together:

30/15

Finally, we divide 30/15 to get 2 as our answer

2

Solve for v

-39v=4v+7(v+7)

Answers

Answer:

-49/50

Step-by-step explanation:

-39v=4v+7(v+7)

Distribute

-39v = 4v +7v+49

Combine like terms

-39v = 11v+49

Subtract 11 v from each side

-39v-11v = 11v-11v +49

-50v = 49

Divide by -50

-50v/-50 = 49/-50

v = -49/50

Answer:

v= - 49/50

Step-by-step explanation:

-39v=4v+7(v+7)

open the parentheses by multiplying both v&7 by 7:

-39v=4v+7v+49

add like terms:

-39v-4v-7v=49

-50v=49

divide both sides by -50:

v= - 49/50

PLEASE HELP!! WILL MARK BRAINLIEST AND THANK YOU!!
Square TUVW with vertices T(-6,1), U(-1,0), V(-2,-5), and W(-7,-4):
(a) Reflection: in the y-axis (b) Translation (x,y) —> (x-5,y+7)
A. (4,7)
B. (-5,7)
C. (1,0)
D. (-4,7)

Answers

Answer:

093006570

Step-by-step explanation:

Rut ectjyvtjyhrjtjtj5j6

write the sentence as an equation.241is the same as 364 fewer than f​

Answers

Answer:

241 = f - 364

Step-by-step explanation:

The answer is 241 = f - 364

Have a nice day / night! :)

pls help again (picture below)

Answers

Answer:

C 10 to the power of 7 equals 10,000,000 and x 3.45 = 3450000000

Step-by-step explanation:

1) What is the VERTEX of the Quadratic Graph below?*

Answers

( 0, -4 ) is the vertex of the parabola

A model with 1 shaded part and 3 unshaded parts. Write the unit fraction depicted in the model as a percent. One-fourth = ?

Answers

Answer: one-fourth = 25%  because= 1/4*100= 1*25= 25%

Hope it Helped! :3

Answer:

1/4=25%

Step-by-step explanation:

Hope this helps

Please help me!!!

Michael's parents doubled his pocket money. His sister paid $1 for ice-cream from her pocket money and he paid $3 of his poket money for an new toy. After making these payments, they have the same money left. If his sister's pocket money is $6, what was Michael's pocket money before doubling it?


1. How much money did they together have after making the payments?

2. How much money did Michael have after his pocket money was doubled. Find the value of x.

3. How much did Micheal's sister have before making the payment?

(ONLY THE ANSWER)

Answers

Answer:

x = $4

1. $10

2. $8

3. $6

Step-by-step explanation:

Let

Michael's doubled pocket money = 2x

He spends $3

Balance = 2x - 3

His sister's pocket money = $6

She spends $1

Balance = $6 - $1

= $5

2x - 3 = 5

2x = 5 + 3

2x = 8

x = 8/2

= 4

x = $4

After making the payments, they both together have $10

After Michael's pocket money was doubled, he has 2x

= 2(4)

= $8

His sister's has $6 before making the payments

-4 = - 17 + x linear equations

Answers

add 17 to both sides

13 = x

hope this helps

See attachment for math work and answer.

A toy company manufactures sealed cubes that contain a colored liquid that glows in the dark. The liquid completely fills the cubes. If 8 cubic blocks with a side length of m are needed to find the volume of each cube, how much of the liquid can each cube hold?

Answers

Answer:

[tex]Volume = 8m^3[/tex]

Step-by-step explanation:

Given

[tex]Cubes = 8[/tex]

[tex]Length = m[/tex] for each

Required

Determine how much all cubes can hold

First, we need to determine the volume of each.

[tex]Volume = m * m * m[/tex]

[tex]Volume = m^3[/tex]

For the 8 cubes, we have:

[tex]Volume = 8 * Volume\ of\ 1[/tex]

[tex]Volume = 8 * m^3[/tex]

[tex]Volume = 8m^3[/tex]

[tex]\frac{x^{2} }{3x^{2}+8x-3 } divided \frac{x-3}{x+2}[/tex]

Answers

Answer:

ddsdvffgdf gddccd fddf

A concert organizer learned from a market survey that when the admission price is $30, there is an average attendance of 800 people. For every $1 drop in price, there is a gain of 20 customers. Each customer spends an average of $5 on concessions. The concert hall has 1,000 seats.

Answers

Answer:

[tex]\mathbf{ P =D(x)= 70 -\dfrac{ x}{20}}[/tex]

Step-by-step explanation:

From the information given :

The objective is to find the demand function which is expressed in p, the price in dollars charged for each ticket, as a function of x, and the number of average attendance is:

Suppose the admission price P = D(x) = 30 and number of average attendant = x = 800 people

Then:

ΔP = -1

Δx = 20

[tex]\left \{ \dfrac{\Delta x}{\Delta P }= -20 }}\ \right.[/tex]

According to point line form :

[tex]\dfrac{\Delta x}{\Delta P }= -20[/tex] where the point is (30, 800)

x - 800 = -20 (P - 30)

x - 800 = -20P + 600

collect like terms

x + 20P = 800+ 600

x + 20P = 1400

x = 1400 - 20P

[tex]\mathsf{ P = 70 -\dfrac{ 1}{20}x}[/tex]

Thus, price in dollars for each ticket P = D(x) is:

[tex]\mathbf{ P =D(x)= 70 -\dfrac{ x}{20}}[/tex]

Other Questions
Help me please hurry??????????? Which correctly lists the three main ocean zones?O deep, neritic, surfaceO intertidal, deep, surfaceO neritic, intertidal, open oceanO open ocean, deep, continental PLEASE HELP IF YOU GET RIGHT I GIVE BRAINLIEST Part A What is a theme of "Stray"? People who work hard are less likely to want to keep a pet. A family's financial situation can affect the decisions it makes. A family should make financial sacrifices for their pets. People should put their children before their pets. Help I need help on this question will mark brainliest:) helppppp -7x + y = -19 -2x+ 3y = -19 solve the system of equations 17+7j=87 What does j equal??????? Sara purchased a life insurance policy as an investment from her neighbor, Angela. Angela, the original policy holder had paid premiums of $12,000 before the sale. Sara paid Angela $16,500 to acquire the life insurance policy. Sara made additional payments of $5,000. When Angela died, Sara collected $50,000. How much of the policy proceeds is taxable to Sara Which scenario is an example of primary succession?A.the formation of soil after a volcanoB.the growth of weeds after a floodC.the establishment of a forest in an abandoned crop fieldD.the recovery of grassland after a fire What kinds of reactions are studied in electrochemistry? A. Oxidation-reduction reactions B. Precipitation reactions C. Decomposition reactions D. Double-replacement reactions(Answer is A) Anthony received $150 for his birthday this year. This was $10 less than half the amount he received last year. How much money did Anthony receive for his birthday last year? write the equation and answer 1/8 divided by 2 3/4 HELP please ASAP Hw for math class 13 hours and 12 minutes. How can you graphically tell if a quadratic equation will have a complex solution? what is 300 = 120,000 there is nothing between the and = cuz its asking me 300 what = 120,000. What is the solution? why is culture important in our society? The Watergate cover-up involved paying each "plumber" how much money to keep quiet? A. $100,000 B. $200,000 C. $300,000 D. $400,000 please help. Free the denominator of the fractionfrom irrationality:[tex] \frac{2}{5 \sqrt{3} } [/tex][tex] \frac{ \sqrt{5 } + 2 }{ \sqrt{5} - 2} [/tex]